« Vrati se
Let n be a positive integer and let (x_1,\ldots,x_n), (y_1,\ldots,y_n) be two sequences of positive real numbers. Suppose (z_2,\ldots,z_{2n}) is a sequence of positive real numbers such that z_{i+j}^2 \geq x_iy_j \qquad for all 1\le i,j \leq n.


Let M=\max\{z_2,\ldots,z_{2n}\}. Prove that

\biggl(\frac{M+z_2+\cdots+z_{2n}}{2n}\biggr)^2\ge \biggl(\frac{x_1+\cdots+x_n}{n}\biggr)\biggl(\frac{y_1+\cdots+y_n}{n}\biggr...

comment
Edited by Orl.

Slični zadaci

#NaslovOznakeRj.KvalitetaTežina
1924IMO Shortlist 1995 problem S40
2016IMO Shortlist 1999 problem A60
2122IMO Shortlist 2003 problem A31
2151IMO Shortlist 2004 problem A513
2238IMO Shortlist 2007 problem A52
2438MEMO 2010 ekipno problem 12